2025 AMC 12A Problems/Problem 15: Difference between revisions
No edit summary |
mNo edit summary |
||
| Line 7: | Line 7: | ||
==Solution 1 (Those who know)== | ==Solution 1 (Those who know)== | ||
This problem is essentially the same as | This problem is essentially the same as [https://artofproblemsolving.com/wiki/index.php?title=2022_AMC_10B_Problems/Problem_14 2022 AMC 10B Problem 14]. | ||
~metrixgo | ~metrixgo | ||
Revision as of 02:37, 8 November 2025
- The following problem is from both the 2025 AMC 10A #21 and 2025 AMC 12A #15, so both problems redirect to this page.
Problem
A set of numbers is called
-
if whenever
and
are (not necessarily distinct) elements of the set,
is not an element of the set. For example,
and the empty set are sum-free, but
is not. What is the greatest possible number of elements in a sum-free subset of
?
Solution 1 (Those who know)
This problem is essentially the same as 2022 AMC 10B Problem 14.
~metrixgo
Solution 2(Quicksolve)
Notice that an odd number plus an odd number always results in an even number. Consider the subset of all odd numbers
. The addition of any even number will result in a violation of the rules, so the maximum number of elements in this subset is 10.
~Kevin Wang
Solution 3
Let
. We are looking for the largest possible sum-free subset
.
First, we show that a sum-free set of size 10 is possible.
Consider the set
. The smallest possible sum of two elements (not necessarily distinct) from this set is
. Since
, no sum of two elements from
can be in
. The size of
is
.
Thus, a sum-free set of size 10 exists.
Next, we prove that no sum-free set
can have 11 or more elements.
Let
be the largest element in
.
Since
is sum-free and
, for any
, we must have
(unless
, in which case
). This is because if both
and
were in
, their sum
would be in
, which is a contradiction.
We can partition the set
into pairs that sum to
, plus any leftover elements.
Case 1:
is even. Let
.
The numbers in
can be grouped as:
along with the numbers
and
.
\item
contains
.
\item
cannot contain
, because
, which is in
.
\item
can contain at most one number from each of the
pairs.
The maximum number of elements
can contain from
is
.
Since
, the maximum size of
is
.
Case 2:
is odd. Let
.
The numbers in
can be grouped as:
along with
.
\item
contains
.
\item
can contain at most one number from each of the
pairs.
The maximum number of elements
can contain from
is
.
Since
, we have
.
The largest possible odd
is 19. If
,
.
So the maximum size of
is
.
In all cases, a sum-free subset of
can have at most 10 elements.
Since we found a set with 10 elements, the greatest possible number of elements is 10.
Video Solution 1 by OmegaLearn
See Also
| 2025 AMC 10A (Problems • Answer Key • Resources) | ||
| Preceded by Problem 20 |
Followed by Problem 22 | |
| 1 • 2 • 3 • 4 • 5 • 6 • 7 • 8 • 9 • 10 • 11 • 12 • 13 • 14 • 15 • 16 • 17 • 18 • 19 • 20 • 21 • 22 • 23 • 24 • 25 | ||
| All AMC 10 Problems and Solutions | ||
| 2025 AMC 12A (Problems • Answer Key • Resources) | |
| Preceded by Problem 14 |
Followed by Problem 16 |
| 1 • 2 • 3 • 4 • 5 • 6 • 7 • 8 • 9 • 10 • 11 • 12 • 13 • 14 • 15 • 16 • 17 • 18 • 19 • 20 • 21 • 22 • 23 • 24 • 25 | |
| All AMC 12 Problems and Solutions | |
These problems are copyrighted © by the Mathematical Association of America.